Show that the two norms in $E=left{fin mathcal{C}^1[0,1]:f(0)=0right}$ are equivalent?












5












$begingroup$


I can't show that the two norms defined as
$$||f||_{infty}=sup_{xin[0,1]}|f(x)+ f'(x)|$$
and
$$N(f)=sup_{xin[0,1]}|f(x)| + sup_{xin[0,1]}|f'(x)|$$
are equivalent in $E={finmathcal{C}^1([0,1]) text{ s.t. } f(0)=0}$.



A first inequality in one sense is trivial.



For the other inequality, I can write:



$$
f(x)=f(0)+int_0^x f'(t) dt
$$

Which implies
$$
sup_{xin[0,1]}|f(x)| leq sup_{xin[0,1]}|f'(x)| .
$$

And then
$$
N(f)leq 2 sup_{xin[0,1]}|f'(x)| .
$$

But I can't conclude from here.



I sincerely thank you for your help.










share|cite|improve this question











$endgroup$












  • $begingroup$
    Thank you for your correction.
    $endgroup$
    – Furdzik
    Jan 5 at 9:58










  • $begingroup$
    @PaulFrost it does not satisfy $f(0)=0$.
    $endgroup$
    – Lorenzo Quarisa
    Jan 5 at 11:07
















5












$begingroup$


I can't show that the two norms defined as
$$||f||_{infty}=sup_{xin[0,1]}|f(x)+ f'(x)|$$
and
$$N(f)=sup_{xin[0,1]}|f(x)| + sup_{xin[0,1]}|f'(x)|$$
are equivalent in $E={finmathcal{C}^1([0,1]) text{ s.t. } f(0)=0}$.



A first inequality in one sense is trivial.



For the other inequality, I can write:



$$
f(x)=f(0)+int_0^x f'(t) dt
$$

Which implies
$$
sup_{xin[0,1]}|f(x)| leq sup_{xin[0,1]}|f'(x)| .
$$

And then
$$
N(f)leq 2 sup_{xin[0,1]}|f'(x)| .
$$

But I can't conclude from here.



I sincerely thank you for your help.










share|cite|improve this question











$endgroup$












  • $begingroup$
    Thank you for your correction.
    $endgroup$
    – Furdzik
    Jan 5 at 9:58










  • $begingroup$
    @PaulFrost it does not satisfy $f(0)=0$.
    $endgroup$
    – Lorenzo Quarisa
    Jan 5 at 11:07














5












5








5


1



$begingroup$


I can't show that the two norms defined as
$$||f||_{infty}=sup_{xin[0,1]}|f(x)+ f'(x)|$$
and
$$N(f)=sup_{xin[0,1]}|f(x)| + sup_{xin[0,1]}|f'(x)|$$
are equivalent in $E={finmathcal{C}^1([0,1]) text{ s.t. } f(0)=0}$.



A first inequality in one sense is trivial.



For the other inequality, I can write:



$$
f(x)=f(0)+int_0^x f'(t) dt
$$

Which implies
$$
sup_{xin[0,1]}|f(x)| leq sup_{xin[0,1]}|f'(x)| .
$$

And then
$$
N(f)leq 2 sup_{xin[0,1]}|f'(x)| .
$$

But I can't conclude from here.



I sincerely thank you for your help.










share|cite|improve this question











$endgroup$




I can't show that the two norms defined as
$$||f||_{infty}=sup_{xin[0,1]}|f(x)+ f'(x)|$$
and
$$N(f)=sup_{xin[0,1]}|f(x)| + sup_{xin[0,1]}|f'(x)|$$
are equivalent in $E={finmathcal{C}^1([0,1]) text{ s.t. } f(0)=0}$.



A first inequality in one sense is trivial.



For the other inequality, I can write:



$$
f(x)=f(0)+int_0^x f'(t) dt
$$

Which implies
$$
sup_{xin[0,1]}|f(x)| leq sup_{xin[0,1]}|f'(x)| .
$$

And then
$$
N(f)leq 2 sup_{xin[0,1]}|f'(x)| .
$$

But I can't conclude from here.



I sincerely thank you for your help.







metric-spaces norm normed-spaces






share|cite|improve this question















share|cite|improve this question













share|cite|improve this question




share|cite|improve this question








edited Jan 5 at 12:33









6005

36.3k751125




36.3k751125










asked Jan 5 at 9:47









FurdzikFurdzik

386




386












  • $begingroup$
    Thank you for your correction.
    $endgroup$
    – Furdzik
    Jan 5 at 9:58










  • $begingroup$
    @PaulFrost it does not satisfy $f(0)=0$.
    $endgroup$
    – Lorenzo Quarisa
    Jan 5 at 11:07


















  • $begingroup$
    Thank you for your correction.
    $endgroup$
    – Furdzik
    Jan 5 at 9:58










  • $begingroup$
    @PaulFrost it does not satisfy $f(0)=0$.
    $endgroup$
    – Lorenzo Quarisa
    Jan 5 at 11:07
















$begingroup$
Thank you for your correction.
$endgroup$
– Furdzik
Jan 5 at 9:58




$begingroup$
Thank you for your correction.
$endgroup$
– Furdzik
Jan 5 at 9:58












$begingroup$
@PaulFrost it does not satisfy $f(0)=0$.
$endgroup$
– Lorenzo Quarisa
Jan 5 at 11:07




$begingroup$
@PaulFrost it does not satisfy $f(0)=0$.
$endgroup$
– Lorenzo Quarisa
Jan 5 at 11:07










2 Answers
2






active

oldest

votes


















2












$begingroup$

Two metrics on the same set are equivalent (i.e. generate the same topology ) iff they have the same set of convergent sequences, because the closure operator in a metric space is entirely and uniquely determined by the convergent sequences.



For brevity let $sup_{xin [0,1]}|g(x)|=|g|_S$ for any continuous $g:[0,1]to Bbb R.$



Idea: Consider that $f(x)+f'(x)=e^{-x}(e^xf(x))'.$



(1). If $lim_{nto infty}|f_n-f|_{infty}=0$:



Then $lim_{nto infty}|e^{-x}(e^x (f_n(x)-f(x))'|_S=0,$ which implies that $lim_{nto infty}|(e^x(f_n(x)-f(x))'|_S=0,$ which, since $f_n(0)=f(0)=0,$ implies that $$lim_{nto infty}|e^x(f_n(x)-f(x))|_S= lim_{nto infty}sup_{xin [0,1]} |int_0^x(e^t(f_n(t)-f(t))'dt,|=0$$ which implies that $$lim_{nto infty}|f_n-f|_S=0.$$ Now $|f'_n-f'|_Sleq |(f_n+f'_n)-(f+f')|_S+|f-f_n)|_S=|f_n-f|_{infty}+|f_n-f|_S, $ so we have $$lim_{nto infty}|f'_n-f'|_S=0.$$ Since $N(f_n-f)=|f_n-f|_S+|f'_n-f'|_S,$ therefore $lim_{nto infty}N(f_n-f)=0.$



(2). If $lim_{nto infty}N(f_n-f)=0$:



Since $N(f_n-f)geq |f_n-f|_{infty}, $ therefore $ lim_{nto infty}|f_n-f|_{infty}=0.$






share|cite|improve this answer











$endgroup$













  • $begingroup$
    Equivalence of norms is stronger than this. One needs to find $m, M > 0$ such that $m N(f) le |f|_infty le MN(f)$ for all $f$.
    $endgroup$
    – Theo Bendit
    Jan 5 at 11:21






  • 1




    $begingroup$
    @TheoBendit. If two norms on a normed linear space are topologically equivalent then they are uniformly equivalent, i.e. $m$ and $M$ exist. Which is an easy exercise: If not, suppose $|v_n|_{(1)}>n|v_n|_{(2)}$ for each $nin Bbb N.$ Let $w_n=frac {v_n}{|v_n|_{(1)}}.$ Then the sequence $(w_n)_{nin Bbb N}$ converges to $0$ with respect to $|cdot|_{(2)}$ but $|w_n|_{(1)}=1$.
    $endgroup$
    – DanielWainfleet
    Jan 5 at 12:19





















3












$begingroup$

As you have already shown, for $f in C^{1}[0,1]$ such that $f(0) = 0$,
$$
sup_{x in [0,1]} |f(x)| le sup_{x in [0,1]} |f'(x)|. tag{1}
$$



We can use this to show that $sup |f|$ is bounded by a constant times $sup |f + f'|$, as follows:
begin{align*}
sup_{x in [0,1]} |f(x)|
&le sup_{x in [0,1]} |e^x f(x)| \
&le sup_{x in [0,1]} left| frac{d}{dx} e^x f(x) right| qquadqquad text{(by (1), since $e^0 f(0) = 0$})\
&= sup_{x in [0,1]} |e^x f(x) + e^x f'(x)| \
&le e sup_{x in [0,1]} |f(x) + f'(x)|.
end{align*}



Now apply triangle inequality to $|f'| = |f' + f - f|$:
begin{align*}
sup |f'(x)| &= sup |f'(x) + f(x) - f(x)| \
&le sup |f(x) + f'(x)| + sup |f(x)| \
&le sup |f(x) + f'(x)| + e cdot sup |f(x) + f'(x)| \
&= (e + 1) sup |f(x) + f'(x)|.
end{align*}



Therefore, $sup |f(x)|$ and $sup |f'(x)|$ are both bounded above by a constant times $sup |f(x) + f'(x)|$. This shows that your two norms are equivalent.






share|cite|improve this answer









$endgroup$













    Your Answer





    StackExchange.ifUsing("editor", function () {
    return StackExchange.using("mathjaxEditing", function () {
    StackExchange.MarkdownEditor.creationCallbacks.add(function (editor, postfix) {
    StackExchange.mathjaxEditing.prepareWmdForMathJax(editor, postfix, [["$", "$"], ["\\(","\\)"]]);
    });
    });
    }, "mathjax-editing");

    StackExchange.ready(function() {
    var channelOptions = {
    tags: "".split(" "),
    id: "69"
    };
    initTagRenderer("".split(" "), "".split(" "), channelOptions);

    StackExchange.using("externalEditor", function() {
    // Have to fire editor after snippets, if snippets enabled
    if (StackExchange.settings.snippets.snippetsEnabled) {
    StackExchange.using("snippets", function() {
    createEditor();
    });
    }
    else {
    createEditor();
    }
    });

    function createEditor() {
    StackExchange.prepareEditor({
    heartbeatType: 'answer',
    autoActivateHeartbeat: false,
    convertImagesToLinks: true,
    noModals: true,
    showLowRepImageUploadWarning: true,
    reputationToPostImages: 10,
    bindNavPrevention: true,
    postfix: "",
    imageUploader: {
    brandingHtml: "Powered by u003ca class="icon-imgur-white" href="https://imgur.com/"u003eu003c/au003e",
    contentPolicyHtml: "User contributions licensed under u003ca href="https://creativecommons.org/licenses/by-sa/3.0/"u003ecc by-sa 3.0 with attribution requiredu003c/au003e u003ca href="https://stackoverflow.com/legal/content-policy"u003e(content policy)u003c/au003e",
    allowUrls: true
    },
    noCode: true, onDemand: true,
    discardSelector: ".discard-answer"
    ,immediatelyShowMarkdownHelp:true
    });


    }
    });














    draft saved

    draft discarded


















    StackExchange.ready(
    function () {
    StackExchange.openid.initPostLogin('.new-post-login', 'https%3a%2f%2fmath.stackexchange.com%2fquestions%2f3062564%2fshow-that-the-two-norms-in-e-left-f-in-mathcalc10-1f0-0-right-are%23new-answer', 'question_page');
    }
    );

    Post as a guest















    Required, but never shown

























    2 Answers
    2






    active

    oldest

    votes








    2 Answers
    2






    active

    oldest

    votes









    active

    oldest

    votes






    active

    oldest

    votes









    2












    $begingroup$

    Two metrics on the same set are equivalent (i.e. generate the same topology ) iff they have the same set of convergent sequences, because the closure operator in a metric space is entirely and uniquely determined by the convergent sequences.



    For brevity let $sup_{xin [0,1]}|g(x)|=|g|_S$ for any continuous $g:[0,1]to Bbb R.$



    Idea: Consider that $f(x)+f'(x)=e^{-x}(e^xf(x))'.$



    (1). If $lim_{nto infty}|f_n-f|_{infty}=0$:



    Then $lim_{nto infty}|e^{-x}(e^x (f_n(x)-f(x))'|_S=0,$ which implies that $lim_{nto infty}|(e^x(f_n(x)-f(x))'|_S=0,$ which, since $f_n(0)=f(0)=0,$ implies that $$lim_{nto infty}|e^x(f_n(x)-f(x))|_S= lim_{nto infty}sup_{xin [0,1]} |int_0^x(e^t(f_n(t)-f(t))'dt,|=0$$ which implies that $$lim_{nto infty}|f_n-f|_S=0.$$ Now $|f'_n-f'|_Sleq |(f_n+f'_n)-(f+f')|_S+|f-f_n)|_S=|f_n-f|_{infty}+|f_n-f|_S, $ so we have $$lim_{nto infty}|f'_n-f'|_S=0.$$ Since $N(f_n-f)=|f_n-f|_S+|f'_n-f'|_S,$ therefore $lim_{nto infty}N(f_n-f)=0.$



    (2). If $lim_{nto infty}N(f_n-f)=0$:



    Since $N(f_n-f)geq |f_n-f|_{infty}, $ therefore $ lim_{nto infty}|f_n-f|_{infty}=0.$






    share|cite|improve this answer











    $endgroup$













    • $begingroup$
      Equivalence of norms is stronger than this. One needs to find $m, M > 0$ such that $m N(f) le |f|_infty le MN(f)$ for all $f$.
      $endgroup$
      – Theo Bendit
      Jan 5 at 11:21






    • 1




      $begingroup$
      @TheoBendit. If two norms on a normed linear space are topologically equivalent then they are uniformly equivalent, i.e. $m$ and $M$ exist. Which is an easy exercise: If not, suppose $|v_n|_{(1)}>n|v_n|_{(2)}$ for each $nin Bbb N.$ Let $w_n=frac {v_n}{|v_n|_{(1)}}.$ Then the sequence $(w_n)_{nin Bbb N}$ converges to $0$ with respect to $|cdot|_{(2)}$ but $|w_n|_{(1)}=1$.
      $endgroup$
      – DanielWainfleet
      Jan 5 at 12:19


















    2












    $begingroup$

    Two metrics on the same set are equivalent (i.e. generate the same topology ) iff they have the same set of convergent sequences, because the closure operator in a metric space is entirely and uniquely determined by the convergent sequences.



    For brevity let $sup_{xin [0,1]}|g(x)|=|g|_S$ for any continuous $g:[0,1]to Bbb R.$



    Idea: Consider that $f(x)+f'(x)=e^{-x}(e^xf(x))'.$



    (1). If $lim_{nto infty}|f_n-f|_{infty}=0$:



    Then $lim_{nto infty}|e^{-x}(e^x (f_n(x)-f(x))'|_S=0,$ which implies that $lim_{nto infty}|(e^x(f_n(x)-f(x))'|_S=0,$ which, since $f_n(0)=f(0)=0,$ implies that $$lim_{nto infty}|e^x(f_n(x)-f(x))|_S= lim_{nto infty}sup_{xin [0,1]} |int_0^x(e^t(f_n(t)-f(t))'dt,|=0$$ which implies that $$lim_{nto infty}|f_n-f|_S=0.$$ Now $|f'_n-f'|_Sleq |(f_n+f'_n)-(f+f')|_S+|f-f_n)|_S=|f_n-f|_{infty}+|f_n-f|_S, $ so we have $$lim_{nto infty}|f'_n-f'|_S=0.$$ Since $N(f_n-f)=|f_n-f|_S+|f'_n-f'|_S,$ therefore $lim_{nto infty}N(f_n-f)=0.$



    (2). If $lim_{nto infty}N(f_n-f)=0$:



    Since $N(f_n-f)geq |f_n-f|_{infty}, $ therefore $ lim_{nto infty}|f_n-f|_{infty}=0.$






    share|cite|improve this answer











    $endgroup$













    • $begingroup$
      Equivalence of norms is stronger than this. One needs to find $m, M > 0$ such that $m N(f) le |f|_infty le MN(f)$ for all $f$.
      $endgroup$
      – Theo Bendit
      Jan 5 at 11:21






    • 1




      $begingroup$
      @TheoBendit. If two norms on a normed linear space are topologically equivalent then they are uniformly equivalent, i.e. $m$ and $M$ exist. Which is an easy exercise: If not, suppose $|v_n|_{(1)}>n|v_n|_{(2)}$ for each $nin Bbb N.$ Let $w_n=frac {v_n}{|v_n|_{(1)}}.$ Then the sequence $(w_n)_{nin Bbb N}$ converges to $0$ with respect to $|cdot|_{(2)}$ but $|w_n|_{(1)}=1$.
      $endgroup$
      – DanielWainfleet
      Jan 5 at 12:19
















    2












    2








    2





    $begingroup$

    Two metrics on the same set are equivalent (i.e. generate the same topology ) iff they have the same set of convergent sequences, because the closure operator in a metric space is entirely and uniquely determined by the convergent sequences.



    For brevity let $sup_{xin [0,1]}|g(x)|=|g|_S$ for any continuous $g:[0,1]to Bbb R.$



    Idea: Consider that $f(x)+f'(x)=e^{-x}(e^xf(x))'.$



    (1). If $lim_{nto infty}|f_n-f|_{infty}=0$:



    Then $lim_{nto infty}|e^{-x}(e^x (f_n(x)-f(x))'|_S=0,$ which implies that $lim_{nto infty}|(e^x(f_n(x)-f(x))'|_S=0,$ which, since $f_n(0)=f(0)=0,$ implies that $$lim_{nto infty}|e^x(f_n(x)-f(x))|_S= lim_{nto infty}sup_{xin [0,1]} |int_0^x(e^t(f_n(t)-f(t))'dt,|=0$$ which implies that $$lim_{nto infty}|f_n-f|_S=0.$$ Now $|f'_n-f'|_Sleq |(f_n+f'_n)-(f+f')|_S+|f-f_n)|_S=|f_n-f|_{infty}+|f_n-f|_S, $ so we have $$lim_{nto infty}|f'_n-f'|_S=0.$$ Since $N(f_n-f)=|f_n-f|_S+|f'_n-f'|_S,$ therefore $lim_{nto infty}N(f_n-f)=0.$



    (2). If $lim_{nto infty}N(f_n-f)=0$:



    Since $N(f_n-f)geq |f_n-f|_{infty}, $ therefore $ lim_{nto infty}|f_n-f|_{infty}=0.$






    share|cite|improve this answer











    $endgroup$



    Two metrics on the same set are equivalent (i.e. generate the same topology ) iff they have the same set of convergent sequences, because the closure operator in a metric space is entirely and uniquely determined by the convergent sequences.



    For brevity let $sup_{xin [0,1]}|g(x)|=|g|_S$ for any continuous $g:[0,1]to Bbb R.$



    Idea: Consider that $f(x)+f'(x)=e^{-x}(e^xf(x))'.$



    (1). If $lim_{nto infty}|f_n-f|_{infty}=0$:



    Then $lim_{nto infty}|e^{-x}(e^x (f_n(x)-f(x))'|_S=0,$ which implies that $lim_{nto infty}|(e^x(f_n(x)-f(x))'|_S=0,$ which, since $f_n(0)=f(0)=0,$ implies that $$lim_{nto infty}|e^x(f_n(x)-f(x))|_S= lim_{nto infty}sup_{xin [0,1]} |int_0^x(e^t(f_n(t)-f(t))'dt,|=0$$ which implies that $$lim_{nto infty}|f_n-f|_S=0.$$ Now $|f'_n-f'|_Sleq |(f_n+f'_n)-(f+f')|_S+|f-f_n)|_S=|f_n-f|_{infty}+|f_n-f|_S, $ so we have $$lim_{nto infty}|f'_n-f'|_S=0.$$ Since $N(f_n-f)=|f_n-f|_S+|f'_n-f'|_S,$ therefore $lim_{nto infty}N(f_n-f)=0.$



    (2). If $lim_{nto infty}N(f_n-f)=0$:



    Since $N(f_n-f)geq |f_n-f|_{infty}, $ therefore $ lim_{nto infty}|f_n-f|_{infty}=0.$







    share|cite|improve this answer














    share|cite|improve this answer



    share|cite|improve this answer








    edited Jan 5 at 12:07

























    answered Jan 5 at 11:16









    DanielWainfleetDanielWainfleet

    35.3k31648




    35.3k31648












    • $begingroup$
      Equivalence of norms is stronger than this. One needs to find $m, M > 0$ such that $m N(f) le |f|_infty le MN(f)$ for all $f$.
      $endgroup$
      – Theo Bendit
      Jan 5 at 11:21






    • 1




      $begingroup$
      @TheoBendit. If two norms on a normed linear space are topologically equivalent then they are uniformly equivalent, i.e. $m$ and $M$ exist. Which is an easy exercise: If not, suppose $|v_n|_{(1)}>n|v_n|_{(2)}$ for each $nin Bbb N.$ Let $w_n=frac {v_n}{|v_n|_{(1)}}.$ Then the sequence $(w_n)_{nin Bbb N}$ converges to $0$ with respect to $|cdot|_{(2)}$ but $|w_n|_{(1)}=1$.
      $endgroup$
      – DanielWainfleet
      Jan 5 at 12:19




















    • $begingroup$
      Equivalence of norms is stronger than this. One needs to find $m, M > 0$ such that $m N(f) le |f|_infty le MN(f)$ for all $f$.
      $endgroup$
      – Theo Bendit
      Jan 5 at 11:21






    • 1




      $begingroup$
      @TheoBendit. If two norms on a normed linear space are topologically equivalent then they are uniformly equivalent, i.e. $m$ and $M$ exist. Which is an easy exercise: If not, suppose $|v_n|_{(1)}>n|v_n|_{(2)}$ for each $nin Bbb N.$ Let $w_n=frac {v_n}{|v_n|_{(1)}}.$ Then the sequence $(w_n)_{nin Bbb N}$ converges to $0$ with respect to $|cdot|_{(2)}$ but $|w_n|_{(1)}=1$.
      $endgroup$
      – DanielWainfleet
      Jan 5 at 12:19


















    $begingroup$
    Equivalence of norms is stronger than this. One needs to find $m, M > 0$ such that $m N(f) le |f|_infty le MN(f)$ for all $f$.
    $endgroup$
    – Theo Bendit
    Jan 5 at 11:21




    $begingroup$
    Equivalence of norms is stronger than this. One needs to find $m, M > 0$ such that $m N(f) le |f|_infty le MN(f)$ for all $f$.
    $endgroup$
    – Theo Bendit
    Jan 5 at 11:21




    1




    1




    $begingroup$
    @TheoBendit. If two norms on a normed linear space are topologically equivalent then they are uniformly equivalent, i.e. $m$ and $M$ exist. Which is an easy exercise: If not, suppose $|v_n|_{(1)}>n|v_n|_{(2)}$ for each $nin Bbb N.$ Let $w_n=frac {v_n}{|v_n|_{(1)}}.$ Then the sequence $(w_n)_{nin Bbb N}$ converges to $0$ with respect to $|cdot|_{(2)}$ but $|w_n|_{(1)}=1$.
    $endgroup$
    – DanielWainfleet
    Jan 5 at 12:19






    $begingroup$
    @TheoBendit. If two norms on a normed linear space are topologically equivalent then they are uniformly equivalent, i.e. $m$ and $M$ exist. Which is an easy exercise: If not, suppose $|v_n|_{(1)}>n|v_n|_{(2)}$ for each $nin Bbb N.$ Let $w_n=frac {v_n}{|v_n|_{(1)}}.$ Then the sequence $(w_n)_{nin Bbb N}$ converges to $0$ with respect to $|cdot|_{(2)}$ but $|w_n|_{(1)}=1$.
    $endgroup$
    – DanielWainfleet
    Jan 5 at 12:19













    3












    $begingroup$

    As you have already shown, for $f in C^{1}[0,1]$ such that $f(0) = 0$,
    $$
    sup_{x in [0,1]} |f(x)| le sup_{x in [0,1]} |f'(x)|. tag{1}
    $$



    We can use this to show that $sup |f|$ is bounded by a constant times $sup |f + f'|$, as follows:
    begin{align*}
    sup_{x in [0,1]} |f(x)|
    &le sup_{x in [0,1]} |e^x f(x)| \
    &le sup_{x in [0,1]} left| frac{d}{dx} e^x f(x) right| qquadqquad text{(by (1), since $e^0 f(0) = 0$})\
    &= sup_{x in [0,1]} |e^x f(x) + e^x f'(x)| \
    &le e sup_{x in [0,1]} |f(x) + f'(x)|.
    end{align*}



    Now apply triangle inequality to $|f'| = |f' + f - f|$:
    begin{align*}
    sup |f'(x)| &= sup |f'(x) + f(x) - f(x)| \
    &le sup |f(x) + f'(x)| + sup |f(x)| \
    &le sup |f(x) + f'(x)| + e cdot sup |f(x) + f'(x)| \
    &= (e + 1) sup |f(x) + f'(x)|.
    end{align*}



    Therefore, $sup |f(x)|$ and $sup |f'(x)|$ are both bounded above by a constant times $sup |f(x) + f'(x)|$. This shows that your two norms are equivalent.






    share|cite|improve this answer









    $endgroup$


















      3












      $begingroup$

      As you have already shown, for $f in C^{1}[0,1]$ such that $f(0) = 0$,
      $$
      sup_{x in [0,1]} |f(x)| le sup_{x in [0,1]} |f'(x)|. tag{1}
      $$



      We can use this to show that $sup |f|$ is bounded by a constant times $sup |f + f'|$, as follows:
      begin{align*}
      sup_{x in [0,1]} |f(x)|
      &le sup_{x in [0,1]} |e^x f(x)| \
      &le sup_{x in [0,1]} left| frac{d}{dx} e^x f(x) right| qquadqquad text{(by (1), since $e^0 f(0) = 0$})\
      &= sup_{x in [0,1]} |e^x f(x) + e^x f'(x)| \
      &le e sup_{x in [0,1]} |f(x) + f'(x)|.
      end{align*}



      Now apply triangle inequality to $|f'| = |f' + f - f|$:
      begin{align*}
      sup |f'(x)| &= sup |f'(x) + f(x) - f(x)| \
      &le sup |f(x) + f'(x)| + sup |f(x)| \
      &le sup |f(x) + f'(x)| + e cdot sup |f(x) + f'(x)| \
      &= (e + 1) sup |f(x) + f'(x)|.
      end{align*}



      Therefore, $sup |f(x)|$ and $sup |f'(x)|$ are both bounded above by a constant times $sup |f(x) + f'(x)|$. This shows that your two norms are equivalent.






      share|cite|improve this answer









      $endgroup$
















        3












        3








        3





        $begingroup$

        As you have already shown, for $f in C^{1}[0,1]$ such that $f(0) = 0$,
        $$
        sup_{x in [0,1]} |f(x)| le sup_{x in [0,1]} |f'(x)|. tag{1}
        $$



        We can use this to show that $sup |f|$ is bounded by a constant times $sup |f + f'|$, as follows:
        begin{align*}
        sup_{x in [0,1]} |f(x)|
        &le sup_{x in [0,1]} |e^x f(x)| \
        &le sup_{x in [0,1]} left| frac{d}{dx} e^x f(x) right| qquadqquad text{(by (1), since $e^0 f(0) = 0$})\
        &= sup_{x in [0,1]} |e^x f(x) + e^x f'(x)| \
        &le e sup_{x in [0,1]} |f(x) + f'(x)|.
        end{align*}



        Now apply triangle inequality to $|f'| = |f' + f - f|$:
        begin{align*}
        sup |f'(x)| &= sup |f'(x) + f(x) - f(x)| \
        &le sup |f(x) + f'(x)| + sup |f(x)| \
        &le sup |f(x) + f'(x)| + e cdot sup |f(x) + f'(x)| \
        &= (e + 1) sup |f(x) + f'(x)|.
        end{align*}



        Therefore, $sup |f(x)|$ and $sup |f'(x)|$ are both bounded above by a constant times $sup |f(x) + f'(x)|$. This shows that your two norms are equivalent.






        share|cite|improve this answer









        $endgroup$



        As you have already shown, for $f in C^{1}[0,1]$ such that $f(0) = 0$,
        $$
        sup_{x in [0,1]} |f(x)| le sup_{x in [0,1]} |f'(x)|. tag{1}
        $$



        We can use this to show that $sup |f|$ is bounded by a constant times $sup |f + f'|$, as follows:
        begin{align*}
        sup_{x in [0,1]} |f(x)|
        &le sup_{x in [0,1]} |e^x f(x)| \
        &le sup_{x in [0,1]} left| frac{d}{dx} e^x f(x) right| qquadqquad text{(by (1), since $e^0 f(0) = 0$})\
        &= sup_{x in [0,1]} |e^x f(x) + e^x f'(x)| \
        &le e sup_{x in [0,1]} |f(x) + f'(x)|.
        end{align*}



        Now apply triangle inequality to $|f'| = |f' + f - f|$:
        begin{align*}
        sup |f'(x)| &= sup |f'(x) + f(x) - f(x)| \
        &le sup |f(x) + f'(x)| + sup |f(x)| \
        &le sup |f(x) + f'(x)| + e cdot sup |f(x) + f'(x)| \
        &= (e + 1) sup |f(x) + f'(x)|.
        end{align*}



        Therefore, $sup |f(x)|$ and $sup |f'(x)|$ are both bounded above by a constant times $sup |f(x) + f'(x)|$. This shows that your two norms are equivalent.







        share|cite|improve this answer












        share|cite|improve this answer



        share|cite|improve this answer










        answered Jan 5 at 12:28









        60056005

        36.3k751125




        36.3k751125






























            draft saved

            draft discarded




















































            Thanks for contributing an answer to Mathematics Stack Exchange!


            • Please be sure to answer the question. Provide details and share your research!

            But avoid



            • Asking for help, clarification, or responding to other answers.

            • Making statements based on opinion; back them up with references or personal experience.


            Use MathJax to format equations. MathJax reference.


            To learn more, see our tips on writing great answers.




            draft saved


            draft discarded














            StackExchange.ready(
            function () {
            StackExchange.openid.initPostLogin('.new-post-login', 'https%3a%2f%2fmath.stackexchange.com%2fquestions%2f3062564%2fshow-that-the-two-norms-in-e-left-f-in-mathcalc10-1f0-0-right-are%23new-answer', 'question_page');
            }
            );

            Post as a guest















            Required, but never shown





















































            Required, but never shown














            Required, but never shown












            Required, but never shown







            Required, but never shown

































            Required, but never shown














            Required, but never shown












            Required, but never shown







            Required, but never shown







            Popular posts from this blog

            Quarter-circle Tiles

            build a pushdown automaton that recognizes the reverse language of a given pushdown automaton?

            Mont Emei